retagged by
695 views

1 Answer

1 votes
1 votes

Given that,

  • $15,{\color{Red}{19}},23,27,31,35,{\color{Red}{39}},\dots, 415 \quad \longrightarrow (1)$
  • $14,{\color{Red}{19}},24,29,34,{\color{Red}{39}}, \dots ,464 \quad \longrightarrow (2)$

Let $’n’$ be a number of common terms in the two sequences.

Common difference in sequence $1 = 4,$ and in sequence $2 = 5.$

$\therefore$ The LCM of $(4,5)=20\quad $ (Common difference of new sequnce)

Now, the sequence will be,

$19,39,59,79,99,\dots , l \left(\leqslant 415\right) \quad \longrightarrow (3)$

We know that, last term $l=a+(n-1)d ,$ where $a = $ first term, $d = $ common difference, $n = $ number of terms.

Now, $l \leqslant 415$

$\Rightarrow 19+(n-1)20 \leqslant 415\quad [\because a  =19,d = 20]$

$\Rightarrow 19+20n-20 \leqslant 415$

$\Rightarrow 20n-1 \leqslant 415$

$\Rightarrow 20n \leqslant 416$

$\Rightarrow n \leqslant \frac{416}{20}$

$ \Rightarrow n \leqslant 20.8$

$\Rightarrow \boxed{n=20}$

Correct Answer: D

Answer:

Related questions

1 votes
1 votes
1 answer
1
go_editor asked Mar 20, 2020
596 views
If $(2n+1)+(2n+3)+(2n+5)+\dots+(2n+47)=5280,$ then what is the value of $1+2+3+\dots+n$ _______
1 votes
1 votes
1 answer
3
1 votes
1 votes
1 answer
4
go_editor asked Mar 20, 2020
495 views
If x is a real number, then $\sqrt{\log _{e}\frac{4x-x^{2}}{3}}$ is a real number if and only if$1\leq x\leq 2$$-3\leq x\leq 3$$1\leq x\leq 3$$-1\leq x\leq 3$
1 votes
1 votes
1 answer
5